Existencia de (V~,A~)(V~,A~)(\tilde{V},\mathbf{\tilde{A}}) para cada (V,A)(V,A)(V,\mathbf {A}) que da lugar a los mismos campos EM

el campo electrico mi y la inducción magnética B se puede parametrizar en términos de potenciales V y A :

mi = V A t , B = × A .
Esta parametrización no es única, ya que podemos encontrar una función escalar θ y definir una pareja ( V ~ , A ~ ) a través de V ~ = V θ / t y A ~ = A + θ ( ) . Entonces ambos ( V , A ) y ( V ~ , A ~ ) dará lugar a la misma ( mi , B ) .

A través de las ecuaciones de Maxwell podemos encontrar un sistema acoplado de ecuaciones diferenciales para V y A :

{ A = m j + ( A + ε m V t ) V = ρ ε t ( A + ε m V t ) , = 2 2 t 2 .

Estos se pueden independizar considerando el calibre de Lorenz, en el que establecemos A + ε m V t = 0 . ¿Cómo se puede mostrar explícitamente que para cada ( V , A ) hay ( V ~ , A ~ ) (es decir, dan lugar a los mismos campos) tales que este par satisfaga la condición de calibre de Lorenz. ¿Es suficiente considerar las expresiones ( ) y deduzca que ambos pares de potenciales deben satisfacer la condición de calibre de Lorenz, lo que da como resultado la condición θ = 0 , es decir, siempre podemos elegir una función escalar θ para cual θ = 0 y considerar un nuevo potencial a través de ( ) ?

Gracias de antemano.

Respuestas (1)

Digamos que tienes un par de potenciales ( V , A ) que no satisfacen el Lorenz Gauge. es decir

A + 1 C 2 V t = F ( X , t ) 0.

Ahora vamos a realizar un cambio de indicador a algunos nuevos ( V , A ) usando la función θ ( X , t ) como lo mencionaste.

A = A + θ V = V θ t

Por supuesto, estos nuevos potenciales han sido construidos de tal manera que también producirán los mismos campos que ( V , A ) , por Invarianza de calibre. Sustituyendo estas relaciones en la ecuación anterior, obtenemos

( A θ ) + 1 C 2 t ( V + θ t ) = F ( X , t ) ,

y reorganizando tenemos

A + 1 C 2 V t 2 θ + 1 C 2 2 θ t 2 = F ( X , t ) .

De esto debe quedar claro que si la función θ ( X , t ) que elegimos cumple la condición

2 θ + 1 C 2 2 θ t 2 = F ( X , t ) ,

entonces

A + 1 C 2 V t = 0.

El problema ahora se reduce a encontrar una función. θ ( X , t ) que resuelve la ecuación de onda anterior con una fuente F ( X , y ) , y de las propiedades de la ecuación de onda, siempre podemos encontrar tal θ ( X , t ) , siempre que sea "fuente" F ( X , t ) no es demasiado loco.


EDITAR: como @hyportnex señala en los comentarios y en esta respuesta , para resolver una ecuación de onda como la anterior, es necesario especificar completamente las condiciones de contorno en θ lo que podría hacer que resolver la ecuación no sea trivial, aunque sigo sintiendo que siempre debería existir una solución. No obstante, agradecería que alguien pudiera corregirme.

Todo esto está bien en el vacío, pero si hay materia presente, también tendrá que satisfacer ciertas condiciones de contorno de continuidad/discontinuidad. No es obvio que esos puedan ser satisfechos por algunos θ dado F con 2 θ + 1 C 2 2 θ t 2 = F ( X , t ) .
@hyportnex Eso es interesante, no lo sabía. quiero decir, claramente θ necesitaría que las condiciones de contorno estuvieran completamente especificadas, pero ¿cómo afecta esto la presencia de carga o corriente en el espacio? ¿Está diciendo que puede haber configuraciones de carga o corriente tales que no se pueda elegir el indicador Lorenz ?
No digo eso, todo lo que digo es que a la ecuación de Helmholtz que usted citó se le deben agregar algunas condiciones de contorno, y no lo veo como una enmienda trivial. consulte, por ejemplo , physics.stackexchange.com/questions/535577/… , pero eso debe hacerse relativistamente invariante.
Muy interesante. Editaré mi respuesta para incluir tu punto.